Equations différentielles

HP == Hors Programme

Calculs

(CCP PC 2001) Correction

Déterminer les fonctions $f:\R\longrightarrow\R$ dérivables vérifiant : $$\forall x\in\R,\quad\quad f'(x)=xf(-x).$$

Correction

Notons d'abord que si $f$ est une solution alors $f$ est continue sur $\R$ puis $f'$ est continue sur $\R$, ce qui donne que $f\in \CC^1(\R)$, puis $f\in \CC^2(\R)$.
En dérivant la relation vérifiée par $f$, on trouve \begin{equation*} \forall x\in \R,~~f''(x)=f(-x)-xf'(-x)\Longrightarrow \forall \in \R^*,~~ f''(x)-\dfrac1x f'(x)-x^2f(x)=0, \end{equation*} Donc $f$ vérifie l'équation différentielle $y''-\frac{1}{x}y'-x^2y=0$.

  1. On vérifie facilement que si $I$ est un intervalle de $\R$ ne contenant pas $0$, alors $$\mathcal{S}_I=\left\{x\in I\longmapsto a\ee^{x^2/2}+b\ee^{-x^2/2},\,~~a,b\in\R\right\}.$$
  2. Soit $y:\R^*\longmapsto\R$ définie par $$y(x)=\left\{ \begin{array}{l} a_1\ee^{x^2/2}+b_1\ee^{-x^2/2},\text{ si } x>0\\ \\ a_2\ee^{x^2/2}+b_2\ee^{-x^2/2},\text{ si } x< 0\\ \end{array}\right.,~~~~a_1,b_1,a_2,b_2\in \R.$$ On vérifie que $y$ peut être raccordé en $0$ par continuité ssi $a_1+b_1=a_2+b_2$, puis pour que la fonction prolongée par continuité en $0$ soit de classe $\CC^2$ en $0$, il faut que $a_1-b_1=a_2-b_2$. Ce qui donne en finale $a_1=a_2$ et $b_1=b_2$.

Conclusion, l'ensemble de solutions sur $\R$ est $$\mathcal{S}_\R=\left\{x\in \R\longmapsto a\ee^{x^2/2}+b\ee^{-x^2/2},\,~~a,b\in\R\right\}.$$

(CCP MP 2004 + $\star$) Correction

Résoudre dans $\R$ l'équation : $\quad\dsp(1+x^2)\,y'-2x\,y =x\exp\left(\frac{1}{1+x^2}\right)$.

Correction

Soit $y$ une solution, on pose $\varphi(x)=\dfrac{y(x)}{1+x^2}$, en dérivant, on trouve, $$\forall x\in\R,~~\varphi'(x)=\dfrac{(1+x^2)y'(x)-2xy(x)}{(1+x^2)^2}=\dfrac{x\exp\left(\frac{1}{1+x^2}\right)}{(1+x^2)^2}$$ Donc, $$\varphi (x)=\int \dfrac{t\exp\left(\frac{1}{1+t^2}\right)}{(1+t^2)^2}\ud t= \lambda -\dfrac{1}{2}\exp\left(\frac{1}{1+x^2}\right)$$ On en déduit, $$y(x)=\lambda (1+x^2)-\dfrac{1+x^2}{2}\exp\left(\frac{1}{1+x^2}\right),\quad \lambda\in \R.$$

($\star\star\star$) Correction

Résoudre l'équation différentielle: $~~x(x+1)y'+y=\Arctan x$

Correction

Soit $y$ une solution de $(E)$. On pose pour $x\neq -1,\,\,g(x)=\dfrac{xy(x)}{(x+1)}$, ensuite on dérive $g$, ce qui donne $$g'(x)=\dfrac{y(x)(1+x)+xy'(x)(1+x)-xy(x)}{(1+x)^2}= \dfrac{x(x+1)y'(x)+y(x)}{(x+1)^2}=\dfrac{\Arctan (x)}{(x+1)^2}$$ Il faut donc chercher une primitive de $x\longmapsto \dfrac{\Arctan (x)}{(x+1)^2}$. On fait une IPP, en posant $u'(x)=\dfrac{1}{(x+1)^2}$ et $v(x)=\Arctan (x)$, on obtient $$ \begin{aligned} \int\dfrac{\Arctan (x)}{(x+1)^2} &=\dfrac{-\Arctan(x)}{x+1}+\int \dfrac{1}{(x+1)(x^2+1)}\\ &\,\\ &=\dfrac{-\Arctan(x)}{x+1}+\int \left(\dfrac{\frac{-1}{2}x+\frac{1}{2}}{(x^2+1)}+\dfrac{\frac{1}{2}}{x+1}\right)\\ &=\dfrac{-\Arctan(x)}{x+1}+\frac{1}{2}\Arctan (x)+\frac{1}{2}\ln (\abs{x+1})-\frac{1}{4}\int\dfrac{2x}{x^2+1}\\ &=\dfrac{-\Arctan(x)}{x+1}+\frac{1}{2}\Arctan (x)+\frac{1}{2}\ln (\abs{x+1})- \frac{1}{4}\ln(x^2+1)+ c(x), \end{aligned} $$ avec $ c(x)=\left\{\begin{array}{ll} a&\text{ si } x\in ]-\infty,-1[\\ b&\text{ si } x\in ]-1,\infty[ \end{array} \right.$
On en déduit que l'ensemble des solutions de l'équation $(E)$ sur un intervalle $I$ tel que $I\subset ]-\infty,-1[$ ou $I\subset ]-1,0[$ ou $I\subset ]0,\infty[$ est $$S =\left\{x\in I\longmapsto -\dfrac{\Arctan(x)}{x}+\frac{(x+1)\Arctan (x)}{2x}+\frac{(x+1)\ln (\abs{x+1})}{2x}-\frac{x+1}{4x}\ln(x^2+1)+\dfrac{a(x+1)}{x},\quad a\in \R\right\}.$$ Raccordement en 0
Soit $f$ une solution sur $]0,\infty[$, alors au voisinage de $0$, on a $$\begin{array}{lcl} f(x)&=& (-1+\dfrac{x^2}{3})+\frac{1}{2}(1+x-\frac{x^3}{3})+\frac{1}{2}(1+x-\frac{x^2}{6})+\frac{1}{4}(x+x^2)+a+\frac{a}{x}+\underset{x\to 0}{\mathrm{o}}(x^2)\\ &=&\frac{a}{x}+a+\frac{x}{2}-\frac{x^2}{6}+\underset{x\to 0}{\mathrm{o}}(x^2) \end{array}$$ Donc pour que $f$ admet une limite finie en $0$ il faut prendre $a=0$, ceci reste valable si $I\subset ]-1,0[$.
On en déduit que l'ensemble des solutions de l'équation $(E)$ sur un intervalle $I$ tel que $I\subset ]-1,\infty[$ est $$S =\left\{f_1\right\} \text{ avec } f_1(x)=-\dfrac{\Arctan(x)}{x}+\frac{(x+1)\Arctan (x)}{2x}+\frac{(x+1)\ln (\abs{x+1})}{2x}-\frac{x+1}{4x}\ln(x^2+1).$$ Raccordement en -1
Soit $f$ une solution sur $]-\infty,-1[$, alors au voisinage de $-1$, on a $$\begin{array} {lcl} f(x)&=& -\dfrac{\Arctan(x)}{x}+\frac{(x+1)\Arctan (x)}{2x}+\frac{(x+1)\ln (\abs{x+1})}{2x}-\frac{x+1}{4x}\ln(x^2+1)+\dfrac{b(x+1)}{x}\quad \quad \quad (b\in \R)\\ &=& \dfrac{-\Arctan(1+h)}{1+h}+\frac{-h\Arctan (1+h)}{2(1+h)}+\frac{h\ln (h)}{2(1+h)}-\frac{h}{4(1+h)}\ln(2-2h+h^2)+\dfrac{bh}{1+h}\quad \quad h=-x-1>0\\ &&\\ &=&\frac{-\pi}{4}-(\frac{1}{2}-\frac{\pi}{8}+\frac{\ln(2)}{4}-b)h+\underset{h\to 0}{\mathrm{o}}(h)+\frac{h\ln (h)}{2(1+h)} \end{array} $$ Ainsi $f$ est prolongeable par continuité en $-1$, pour tout valeur de $b$. De plus, on a $$\lim_{x\to (-1)^{-}}f(x)= \dfrac{-\pi}{4}=\lim_{x\to (-1)^+} f_1(x)$$ Il faut maintenant regarder la dérivabilité de $f$ en -1!
La fonction $f$ ne peut pas être dérivable en $x=-1$ puisque $$h=-x-1>0,\quad \dfrac{f(-1+h)-\frac{-\pi}{4}}{h}= -(\frac{1}{2}-\frac{\pi}{8}+\frac{\ln(2)}{4}-b)+\underset{h\to 0}{\mathrm{o}}(1)+\frac{\ln (h)}{2(1+h)}$$ n'a pas de limite lorsque $h$ tendvers $0$.


Courbe représentative de $f$

(CCP PSI 2009) HP Correction

Trouver toutes les solutions sur $\R$ de l'équation différentielle $~~(x+1)y'' - y' -xy = \ee^{-x}\quad (E)$. On pourra remarquer que $x\longmapsto \ee^x$ est solution de l'équation homogène.

Correction

Soit $I$ un intervalle de $\R$ ne contenant pas $-1$. On note $f(x)=\ee^x$, on cherche une deuxième solution $g$ telle que $(f,g)$ soit un système fondamental de solution de $y''-\frac{1}{x+1}y'-\frac{x}{x+1}y=0$.

  1. On note $W$ le Wronskien de $f$ et $g$, alors $W$ vérifie, $(1+x)W'-W=0$, on peut donc prendre $W(x)=x+1$.
  2. On définie $\varphi=g/f$, donc $$\forall \in I,~~\varphi'(x)=\dfrac{W(x)}{f(x)^2}=\dfrac{x+1}{\ee^{2x}}\Longrightarrow\varphi(x)=\dfrac{3+2x}{-4}\ee^{-2x}.$$
  3. On peut prendre alors $g(x)=(3+2x)\ee^{-x}$.
  4. On remarque (après un peu de calculs), que $x\longmapsto \ee^{-x}/2$ est une solution particulière de l'équation $(E)$.

Donc, $$S_I=\left\{x\longmapsto \dfrac{\ee^{-x}}{2}+a\ee^x+b(3+2x)\ee^{-x},\,\,a,b\in \R\right\}.$$ Solutions sur $\R$
Soit $y$ une fonction définie par $$y(x)=\left\{\begin{array}{l} \dfrac{\ee^{-x}}{2}+a_1\ee^x+b_1(3+2x)\ee^{-x},\text{ si } x< -1\\ \\ \dfrac{\ee^{-x}}{2}+a_2\ee^x+b_2(3+2x)\ee^{-x},\text{ si } x>-1 \end{array} \right.,\quad a_1,b_1,a_2,b_2\in \R.$$ On a $\dsp\limiteX{x}{-1^-}y(x)=(1/2+b_1)\ee+\frac{a_1}{\ee}$ et $\dsp\limiteX{x}{-1^+}y(x)=(1/2+b_2)\ee+\frac{a_2}{\ee}$, donc on doit avoir $$(1/2+b_1)\ee+\frac{a_1}{\ee}=(1/2+b_2)\ee+\frac{a_2}{\ee}\Longrightarrow\ee^2 b_1+a_1=\ee^2 b_2+a_2.$$ La dérivabilité de $y$ en $-1$ donne la même condition.
La condition que $y$ soit deux fois dérivable en $-1$, donne $a_1-3b_1\ee^2=a_2-3b_2\ee^2$. En combinant les deux relations on trouve $a_1=a_2$ et $b_1=b_2$.
Conclusion $$S_\R=\left\{x\longmapsto \dfrac{\ee^{-x}}{2}+a\ee^x+b(3+2x)\ee^{-x},\,\,a,b\in \R\right\}.$$

($\star\star$) Correction

Soit $m \in \R$. Déterminer la solution de l'équation : $$ (E_m) \qquad y'' - 2y' + (1+m^2)y = (1+4m^2)\cos{mx}$$ qui vérifie $y(0)=1$ et $y^\prime (0)=0$ (Indication : On traitera séparément les cas $ m=0$ et $ m \not = 0$).

Correction

On distingue deux cas:

  1. $m\neq 0$. Dans ce cas l'équation caractéristique est $x^2-2x+(1+m^2)=0$ dont les racines sont $\lambda_1=1+\ii m$ et $\lambda_2=1-\ii m$. On en déduit alors: $$S_H=\{x\longmapsto \ee^x (a\cos(mx)+b\sin(mx)),\,\,a,b\in \R\}.$$ On cherche une solution particulière sous la forme $f(x)=a\cos(mx)+ b\sin (mx)$, ce qui donne: $$\left\{\begin{array}{l} f'(x)=-am\sin(mx)+bm\cos(mx)\\ f''(x)=-am^2\cos(mx)-bm^2\sin(mx) \end{array}\right.$$ soit $$f''(x)-2f'(x)+(1+m^2)f(x)=(a-2bm)\cos(mx)+(b+2am)\sin(x) $$ Par identification, on trouve $a=1,\,b=-2m$, finalement l'ensemble des solutions est: $$S=\left\{x\longmapsto \ee^x (a\cos(mx)+b\sin(mx))+\cos(mx)-2m\sin(mx),\,\,a,b\in \R\right\}.$$ Soit $f\in S$ telle que $f(0)=1$ et $f'(0)=0$, ceci nous donne, $$\left\{ \begin{array}{lcl} a+1&=&1\\ a+bm-2m^2&=&0 \end{array} \right. \Longrightarrow a=0 ,\,b=2m $$ donc $$ \boxed{f(x)= 2m\sin(mx)\left(\ee^{x}-1\right)+\cos(mx)}.$$
  2. $m=0$. Dans ce cas $f(x)=1$ est la solution du problème de Cauchy proposé.

($\star$) Correction

Soit $\alpha\in \R$. Déterminer les solutions réelles des équations différentielles suivantes: $$\mathbf{1.\,} y''-2\alpha y'+y=\ee^{\alpha x}(x+1),\quad \mathbf{2.\,} y''-3y+\alpha y=\sin(x), \quad\mathbf{3.\,} y''-2\alpha y+(\alpha^2+1)y=\ee^x\sin(x).$$

Correction

  1. En fonction de $\alpha$, on a 3 cas:
    1. $\abs{\alpha}>1$, $S=\left\{x\longmapsto \ee^{\alpha x}\left(a\ch\left( x\sqrt{\alpha^2 -1}\right) +b\sh\left(x\sqrt{\alpha^2-1}\right)+\dfrac{x+1}{1-\alpha^2}\right),\,\,a,b\in \R\right\}$.
    2. $\abs{\alpha}< 1$, $S=\left\{x\longmapsto \ee^{\alpha x}\left(a\cos\left( x\sqrt{1-\alpha^2 }\right) +b\sin\left(x\sqrt{1-\alpha^2}\right)+\dfrac{x+1}{1-\alpha^2}\right),\,\,a,b\in \R\right\}$.
    3. $\abs{\alpha}=1$, $S=\left\{x\longmapsto \ee^{\alpha x}\left(ax +b+\dfrac{x^3}{6}+\dfrac{x^2}{2}\right),\,\,a,b\in \R\right\}$.
  2. En fonction de $\alpha$, on a 3 cas:
    1. $\alpha=\dfrac{9}{4}$, $S=\left\{x\longmapsto \ee^{\frac{3}{2} x}\left(a x +b\right)+\dfrac{1} {(\alpha-1)^2+9}\left((\alpha-1)\sin(x)+3\cos(x)\right),\,\,a,b\in \R\right\}$.
    2. $\alpha< \dfrac{9}{4}$, $$S=\left\{x\longmapsto a\ee^{\alpha_1 x} +b\ee^{\alpha_2 x}+\dfrac{1}{(\alpha-1)^2+9} \left((\alpha-1)\sin(x)+3\cos(x)\right),\,\,a,b\in \R\right\},$$ avec $\alpha_1,\alpha_2$ sont les racines réelles de $x^2-3x+\alpha=0$.
    3. $\alpha>\dfrac{9}{4}$, $$S=\left\{x\longmapsto (a\cos(\beta x)+b\sin(\beta x))\ee^{\frac{3}{2} x} +\dfrac{1} {(\alpha-1)^2+9}\left((\alpha-1)\sin(x)+3\cos(x)\right),\,\,a,b\in \R\right\},$$ avec $\beta =\sqrt{\alpha-\frac{9}{4}}$.
  3. On distingue deux cas:
    1. $\alpha =1$, $S=\left\{x\longmapsto \ee^x\left(a\cos(x)+b\sin(x)\right)-\dfrac{\ee^x}{2}\left(x\cos(x)-\sin(x)\right),\,\,a,b\in \R\right\}$.
    2. $\alpha \neq 1$, $S=\left\{x\longmapsto \ee^{\alpha x}\left(a\cos(x)+b\sin(x)\right)-\dfrac{\ee^x}{\alpha^3-3\alpha^2+7\alpha-5}\left(2\cos(x)+(\alpha-1)\sin(x)\right),\,\,a,b\in \R\right\}$.

Changement de variable

($\star$) Correction

Résoudre l'équation différentielle $(1+x^2)y''+xy'+k^2y=0, \,\,(k\in \R)$, à l'aide du changement de variable $x=\sh(t)$.

Correction

La fonction $\fonct{\tan}{\R}{\R}{t}{\sh(t)}$ est $\CC^2$-difféomorphisme de $\R$ dans $\R$ (dont l'application réciproque est $\Argsh(x)=\ln(x+\sqrt{x^2+1})$).
Soit $y$ une solution de $E$, on pose, pour $t\in \R$, $f(t)=y(\sh(t))$, $f$ est donc de classe $\CC^2$ sur $\R$ et de plus, on a $$\begin{array} {lcl}\forall t\in \R,\,\, f'(t)&=&y'(\sh(t))\ch(t),\,\,\\ f''(t)&=&y''(\sh(t))\ch^2(t)+y'(\sh(t))\sh(t)\\ &=&y''(\sh(t))(1+\sh(t)^2)+y'(\sh(t))(t)\sh(t)\\ &=&-k^2y(\sh(t)). \end{array} $$ Comme $y$ est une solution de $E$, on en déduit que $f$ est une solution de l'équation $f''+k^2f=0$, soit $f(t)=a\cos(kt)+b\sin(k)$ donc $$ \forall x\in \R,\,y(x)=a \cos\left(k\ln\left(x+\sqrt{x^2+1}\right)\right)+b\sin\left(k\ln\left(x+\sqrt{x^2+1}\right)\right).$$

($\star$) Correction

Résoudre l'équation différentielle $(1+x^2)^2y''+2x(1+x^2)y'+y=0$, à l'aide du changement de variable $x=\tan (t)$.

Correction

La fonction $\fonct{\tan}{]-\pi/2,\pi/2[}{\R}{t}{\tan(t)}$ est $\CC^2$-difféomorphisme de $]-\pi/2,\pi/2[$ dans $\R$ (dont l'application réciproque est $\Arctan$).
Soit $y$ une solution de $E$, on pose, pour $t\in \left]-\frac{\pi}{2},\frac{\pi}{2}\right[$, $f(t)=y(\tan(t))$, $f$ est donc de classe $\CC^2$ sur $\left]-\frac{\pi}{2},\frac{\pi}{2}\right[$ et de plus, on a $$\begin{array} {lcl} \forall t\in \left]-\frac{\pi}{2},\frac{\pi}{2}\right[,\,\, f'(t)&=&y'(\tan(t))(1+\tan^2(t)),\,\,\\ f''(t)&=&y''(\tan(t))\left(1+\tan^2(t)\right)^2+2y'(\tan(t))\tan(t)(1+\tan^2(t)). \end{array} $$ Comme $y$ est une solution de $E$, on en déduit que $f$ est une solution de l'équation $f''+f=0$, soit $$f(t)=a\cos(t)+b\sin(t),\quad a,b\in \R,$$ donc, $$\forall x\in \R,\,y(x)=a\cos(\Arctan(x))+b\sin(\Arctan(x))=\dfrac{a}{\sqrt{1+x^2}}+\dfrac{bx}{\sqrt{1+x^2}}.$$

($\star$) Correction

Résoudre l'équation différentielle $xy''-y'-4x^3y=0$ sur $\R_+^*$, à l'aide du changement de variable $x=\sqrt{t}$.

Correction

Soit $y$ une solution de l'équation différentielle, on pose $f(t)=y(\sqrt{t})$ (la fonction $t\longmapsto \sqrt{t}$ est $\CC^2$-difféomorphisme de $\R_+^*$ dans lui même). On a alors, $f\in \CC^2(\R_+^*,\R)$ et pour tout $t>0$, $f'(t)=\dfrac{y'(\sqrt{t})}{2\sqrt{t}}$ et $$f''(t)=\dfrac{y''(t)}{4t}- \dfrac{y'(\sqrt{t})}{4t\sqrt{t}}=\dfrac{1}{4t\sqrt{t}}\left(\sqrt{t}y''(\sqrt{t})-y'(\sqrt{t})\right) \underset{y \text{ sol de }E}{=}\dfrac{4(\sqrt{t})^3y(\sqrt{t})}{4t\sqrt{t}}=f(t).$$ On en déduit que $f$ est une solution de l'équation $f''-f=0$ donc $$\forall t>0,\quad f(t)=a\ee^t+b\ee^{-t},\,a,b$$ soit finalement, $$\forall x>0,\,\,y(x)=a\ee^{x^2}+b\ee^{-x^2},\,a,b\in \R.$$

($\star\star$) Correction

Résoudre sur $\R_+^*$ l'équation $x^2y''+y=0$ (on pourrait chercher des solutions sous la forme $y(x)=x^a$). En déduire les fonctions de classe $\CC^1$ sur $\R_+$ telles que $f'(x)=f(1/x)$.

Correction

Soit $a\in \C$, pour $x>0$, on pose $y(x)=\ee^{a\ln(x)}$ alors $y$ est une solution de $(E)$ ssi $a^2-a+1=0$ soit $a\in \{\frac{1+\ii \sqrt{3}}{2},\,\frac{1-\ii \sqrt{3}}{2}\}$.
Posons alors, pour $x>0$, $$\begin{array} {lcl} y_1(x)&=&\ee^{\frac{1+\ii \sqrt{3}}{2}\ln(x)}=\sqrt{x}\left(\cos\left(\frac{\sqrt{3}}{2}\ln(x)\right)+\ii\sin\left(\frac{\sqrt{3}}{2}\ln(x)\right)\right),\\ &&\\ y_2(x)&=&\ee^{\frac{1-\ii \sqrt{3}}{2}\ln(x)}=\sqrt{x}\left(\cos\left(\frac{\sqrt{3}}{2}\ln(x)\right)-\ii\sin\left(\frac{\sqrt{3}}{2}\ln(x)\right)\right). \end{array} $$ Ces deux solutions forment un système fondamental de solutions sur $\C$, pour obtenir les solutions réelles, on pose, pour tout $x>0$: $$\begin{array} {lcl} f_1(x)&=&\dfrac{y_1(x)+y_2(x)}{2}= \sqrt{x}\cos\left(\frac{\sqrt{3}}{2}\ln(x)\right),\\ f_2(x)&=&\dfrac{y_1(x)-y_2(x)}{2\ii}=\sqrt{x}\sin\left(\frac{\sqrt{3}}{2}\ln(x)\right). \end{array} $$ On en déduit que les solution de l'équation $(E)$ sont de la forme $x>0\longmapsto Af_1(x)+Bf_2(x),\,\,A,B\in \R$.
Soit $f\in \CC^1(\R_+^*)$ telle que, pour tout $x>0$, on a $f'(x)=f(1/x)$, alors cette relation montre que $f'$ est de classe $\CC^1$ comme la composée de deux fonctions de classe $\CC^1$, donc $f$ est de classe $\CC^2$ sur $\R_+^*$. On a alors, $$\forall x>0,\,\,f''(x)=-f'\left(\frac{1}{x}\right)\frac{1}{x^2}=\dfrac{-f(x)}{x^2} \Longrightarrow \forall x>0,\, x^2f''(x)+f(x)=0.$$ Il existe alors $A,B\in \R$ tes que $f(x)=\sqrt{x}\left(A\cos\left(\frac{\sqrt{3}}{2}\ln(x)\right)+B\sin\left(\frac{\sqrt{3}}{2}\ln(x)\right)\right)$, en remplaçant dans l'équation initiale, on obtient: $$\begin{array} {lcl} f'(x)&=&\dfrac{1}{2\sqrt{x}}\left(\left(A+B\sqrt{3}\right)\cos\left(\frac{\sqrt{3}}{2}\ln(x)\right)+\left(B-A\sqrt{3}\right)\sin\left(\frac{\sqrt{3}}{2}\ln(x)\right)\right)\\ &=&\dfrac{1}{\sqrt{x}}\left(A\cos\left(\frac{\sqrt{3}}{2}\ln(x)\right)-B\sin\left(\frac{\sqrt{3}}{2}\ln(x)\right)\right) \end{array} $$ Ce qui donne $A=B\sqrt{3}$, donc $$f(x)=B\sqrt{x}\left(\sqrt{3}\cos\left(\frac{\sqrt{3}}{2}\ln(x)\right)+\sin\left(\frac{\sqrt{3}}{2}\ln(x)\right)\right)=2B\sqrt{x}\cos\left(\frac{\sqrt{3}}{2}\ln(x)-\frac{\pi}{6}\right).$$

($\star$) Correction

Soit $a\in \R$. Résoudre l'équation différentielle $y^{(3)}+y''+y'+y=\ee^{ax}$.

Correction

Soit $y$ une solution de $E$, on pose $z=y+y'$ alors $z$ est de classe $\CC^2$ (puisque $y$ est de classe $\CC^3$) et on a $z'=y'+y''$, $z''=y''+y^{(3)}$ donc $z$ est une solution de l'équation $z''+z=\ee^{ax}$ dont l'ensemble de solutions est $$S_1=\left\{x\longmapsto A\cos(x)+B\sin(x)+ \dfrac{\ee^{ax}}{1+a^2},\,\,A,B\in \R\right\}.$$ Retour à $y$:
Les solutions de $E$ alors vérifient l'équation $E_1$ suivantes: $y'+y=A\cos(x)+B\sin(x)+\dfrac{1}{1+a^2}\ee^{ax}$, d'où $$\left\{\begin{array}{lcl} y(x)=C\ee^{-x}+\dfrac{A-B}{2}\cos(x)+\dfrac{A+B}{2} \sin(x)+\dfrac{1}{(1+a)(1+a^2)}\ee^{ax},\,\,C,A,B\in \R,&\text{ si } a\neq -1\\ &&\\ y(x)=C\ee^{-x}+\dfrac{A-B}{2}\cos(x)+\dfrac{A+B}{2} \sin(x)+\dfrac{x}{2}\ee^{ax},\,\,C,A,B\in \R,&\text{ si } a= -1 \end{array} \right.$$

($\star$) Correction

Résoudre sur $\R_+^*$ l'équation différentielle: $$\forall x\in \R_+^*,\,\,x^2y''(x)-2xy'(x)+(2-x^2)y(x)=0.$$ On pourrait poser $y(x)=xz(x)$.

Correction

L'application $s:x\mapsto \frac{1}{x}$ est de classe $\CC^2$ sur $\R_+^*$, de plus $y$ l'est aussi, il en résulte que $z=sy$ est de classe $\CC^2$ sur $\R_+^*$ comme le produit de deux fonction de classe $\CC^2$.
D'autre par, en utilisant la règle de dérivation de produit, on trouve: $$\forall x>0,\,y'(x)=xz'(x)+z(x),\,\,y''(x)=xz''(x)+2z'(x).$$ Comme $y$ est une solution de l'équation (E), en remplaçant dans cette équation, on trouve, pour tout $x>0$: $$x^3z''(x)+2x^2z'(x)-2x^2z'(x)-2xz(x)+2xz(x)-x^3z(x)=0\Longleftrightarrow \forall x>0,\, x^3(z''(x)-z(x))=0.$$ Puisque $x\neq 0$, on en déduit alors que $z$ vérifie l'équation (E'): $z''-z=0$.
L'ensemble des solutions de l'équation (E') est $S_1=\{x\mapsto a\ee^x+b\ee^{-x},\,a,b\in \R\}$. Notons $S$ l'ensemble des solutions de l'équation (E). On a montré que: $$y\in S\Longleftrightarrow x\mapsto \frac{y(x)}{x}\in S_1,$$ On en déduit alors que $S=\{x\mapsto ax\ee^{x}+bx\ee^{-x},\,a,b\in \}$.

($\star$}) Correction

Résoudre $x^2y''-xy'+y=x\,\ln x$ à l'aide du changement de variable $t=\ln \abs{x}$.

Correction

On cherche une solution de cette équation différentielle sur $\R_+^*$.
On fait le changement de variable $t=\ln(x)$, soit $y$ une solution de l'équation, on définie $z$ par la relation $z(t)=z(\ln(x))=y(x)$ ( ou $z(t)=y(\ee^t)$). Ce qui donne $$\left\{\begin{array}{l} y(x)=z(t)\\ y'(x)=z'(t)\dfrac{\ud t}{\ud x}=z'(t)\dfrac{1}{x}\\ y''(x)=z''(t)\dfrac{1}{x^2}-z'(t)\dfrac{1}{x^2} \end{array} \right. \Longrightarrow z''-2z'+z=t\ee^t.$$ La résolution de l'équation vérifiée par $z$, nous donne $$\mathcal{S}_z=\left\{t\longmapsto (a+bt)\ee^t+\dfrac{t^3\ee^t}{6},\,\,a,b\in \R\right\}.$$ Donc les solutions de l'équation différentielle initiale sur $]0,\infty[$ sont $$\mathcal{S}=\left\{x\longmapsto (a+b\ln(x))x+\dfrac{x\ln(x)^3}{6},\,\,a,b\in \R\right\}.$$

(CCP PC 2006) Correction

On considère l'équation $(\mathscr{E}): \quad 4x^2y''-8xy'+9y=x^2+1$.

  1. Chercher une solution polynomiale de $(\mathscr{E})$ sur~$\R$.
  2. Solutions de~$E$ sur $]0,+\infty[$ ? On pourra poser $Y(t)=y(\ee^t)$.
  3. Solutions de~$E$ sur $]-\infty,0[$ ? On pourra poser $Y(t)=y(-\ee^t)$.
  4. Solutions sur $\R$ ?

Correction

  1. Une solution polynomiale est $y_0(x)=\dfrac19+x^2$.
  2. On pose $Y(t)=y(\ee^t)$. Alors $$Y'(t)=\ee^t\,y'(\ee^t)=x\,y'(x),\quad Y''(t)=\ee^t\,y'(\ee^{t})+ \ee^{2t}\,y''(\ee^t)=x\,y'(x)+x^2\,y''(x).$$ On en déduit une équation linéaire à coefficients constants en~$Y$ : $4\,Y''-12\,Y'+9\,Y=0$, dont l'équation caractéristique admet $\alpha=3/2$ comme racine double. Cela donne, après résolution, les solutions sur~$\R_+^{*}$, $$y(x)= (\lambda+\mu\ln(x))\,x^{3/2} + \dfrac19+x^2,\quad \lambda,\mu\in \R$$
  3. De la même façon, sur $\R_-^{*}$, on trouve la même équation à coefficients constants et donc les solutions sur~$\R_+^{*}$, $$y(x)= (\lambda'+\mu'\ln(-x))\,(-x)^{3/2} + \dfrac19+x^2,\quad \lambda',\mu'\in \R$$
  4. La continuité et la classe~$\CC^1$ ne donnent rien ; en revanche, la dérivabilité deux fois imposent la nullité des quatre coefficients inconnus. Ainsi, la seule solution sur $\R$ est $x\mapsto x^2+\dfrac19$.

($\star$) Correction

Résoudre $~~x^3y''-2xy+3=0$ (On pourrait poser $z=xy'+y$).

Correction

Soit $I$ un intervalle de $\R$ qui ne contient pas $0$. Soit $y$ une solution de $(\EE)$ sur $I$, on pose, pour $x\in I,~~z(x)=xy'(x)+y(x)$.

  1. On vérifie que $z$ est solution de l'équation $(\EE_1)~~xz'-2z=\dfrac{-3}{x}$, dont l'ensemble de solution $$S_1=\left\{ x\in I\longmapsto \dfrac{1}{x}+ax^2,~~a\in \R\right\}.$$
  2. On en déduit que $y$ vérifie $xy'+y= \dfrac{1}{x}+ax^2$, ce qui donne $$S=\left\{x\in I\longmapsto \dfrac{\ln(\abs{x})}{x}+\dfrac{ax^2}{3}+\dfrac{b}{x},~~a,\,b\in \R\right\}.$$

Enfin, aucune solution de $(\EE)$ ne peut être prolonger par continuité en $0$, donc $(\EE)$ ne pas de solutions définie sur $\R$.

Sérires

($\star\star$) Correction

On considère l'équation différentielle $(E):\,xy''+3y'-4x^3y=0$. Déterminer les solutions de $(E)$ développables en séries entières.

Correction

Soit $f(x)=\dsp\sum\limits_{n\geq 0}a_nx^n$ une somme de SE de rayon de convergence $R\geq 0$, supposons que $f$ est une solution de $E$, alors $\forall x\in ]-R,R[,\,f'(x)=\dsp\sum\limits_{n\geq 1}na_n x^{n-1},\, f''(x)=\dsp\sum\limits_{n\geq 2}n(n-1)a_nx^{n-2}$ ce qui donne, pour tout $x\in ]-R,R[$ $$\begin{array} {lcl} xf''(x)+3f'(x)-4x^3f(x)&=&\dsp\sum\limits_{n\geq 1}\left(n+2\right)na_nx^{n-1}-\dsp\sum\limits_{n\geq 0}4a_nx^{n+3}\\ &=& \dsp\sum\limits_{n\geq 0}\left(n+3\right)(n+1)a_{n+1}x^{n}-\dsp\sum\limits_{n\geq 3}4a_{n-3}x^{n} \end{array} $$ Donc $f$ est une solution ssi pour tout $x\in ]-R,R[$ $$0=3a_1+8a_2x+15a_3x^2+\dsp\sum\limits_{n\geq 3}\left((n+3)(n+1)a_{n+1}-4a_{n-3}\right)x^{n}$$ Ce qui implique $$\left\{\begin{array}{l} a_1=a_2=a_3=0\\ \forall n\geq 3,\,\, a_{n+1}=\dfrac{4a_{n-3}}{(n+3)(n+1)}. \end{array}\right. $$ Ceci nous donne, pour tout $k\in \N$, $$\begin{array} {lcl} a_{4k+3}&=&a_{4k+2}=a_{4k+1}=0,\,\,\\ a_{4k+4}&=&\dfrac{4a_{4k}}{(4k+6)(4k+4)}=\dfrac{a_{4k}}{(2k+3)(2k+2)}=\dfrac{a_0}{(2k+3)!}. \end{array} $$ Ces relations se démontrent par récurrence sur $k\in \N$.
On en déduit alors que $f(x)=a_0\dsp\sum\limits_{n\geq 0}\dfrac{x^{4n}}{(2n+1)!}$, un calcul simple montre que $R=\infty$. D'autre part, $$\forall x\neq 0,\, \,f(x)=\dfrac{1}{x^2}\dsp\sum\limits_{n\geq 0} \dfrac{x^{4n+2}}{(2n+1)!}=a_0\dfrac{1}{x^2}\dsp\sum\limits_{n\geq 0} \dfrac{(x^2)^{2n+1}}{(2n+1)!}=a_0\dfrac{\sh(x^2)}{x^2}.$$ Posons alors $f(x)=\left\{\begin{array}{lcl} \dfrac{\sh(x^2)}{x^2}&\text{ si }&x\neq 0\\ 1&\text{ si }& x=0 \end{array} \right.$. Alors l'ensemble des solutions DSE de $(E)$ est $\Vect (f)$.

($\star\star$) Correction

On considère l'équation différentielle $(E):\,xy''+2y'+xy=0$. Déterminer les solutions de $(E)$ développables en séries entières.

Correction

Soit $f(x)=\dsp\sum\limits_{n\geq 0}a_nx^n$ une somme de SE de rayon de convergence $R\geq 0$, supposons que $f$ est une solution de $E$, alors $\forall x\in ]-R,R[,\,f'(x)=\dsp\sum\limits_{n\geq 1}na_nx^{n-1},\,f''(x)=\dsp\sum\limits_{n\geq 2}n(n-1)a_nx^{n-2}$ ce qui donne, $\forall x\in ]-R,R[,\,\,$ $$\begin{array} {lcl} xf''(x)+2f'(x)+xf(x)&=&\dsp\sum\limits_{n\geq 1}\left(n+1\right)na_nx^{n-1}+ \dsp\sum\limits_{n\geq 0}a_nx^{n+1}\\ &= &\dsp\sum\limits_{n\geq 0}\left(n+1\right)(n+2)a_{n+1}x^{n}+\dsp\sum\limits_{n\geq 1}a_{n-1}x^{n} \end{array} $$ Donc $f$ est une solution ssi pour tout $x\in ]-R,R[$ $$0=2a_1+\dsp\sum\limits_{n\geq 1}\left((n+2)(n+1)a_{n+1}+a_{n-1}\right)x^{n}\Longrightarrow\left\{\begin{array}{l} a_1=0\\ \forall n\geq 1,\,\, a_{n+1}=\dfrac{-a_{n-1}}{(n+2)(n+1)}. \end{array}\right. $$ Ceci nous donne, en utilisant l'unicité de DSE, $$\forall k\in \N,\,a_{2k+1}=0,\,\,a_{2k+2}=\dfrac{-a_{2k}}{(2k+4)(2k+3)}=\dfrac{(-1)^ka_0}{(2k+3)!}.$$ On en déduit alors que $f(x)=a_0\dsp\sum\limits_{n\geq 0}(-1)^n\dfrac{x^{2n}}{(2n+1)!}$, un calcul simple montre que $R=\infty$. D'autre part, $$\forall x\neq 0,\, \,f(x)=\dfrac{1}{x}\dsp\sum\limits_{n\geq 0} (-1)^n\dfrac{x^{2n+1}}{(2n+1)!}=a_0\dfrac{\sin(x)}{x}.$$ Posons alors $f(x)=\left\{\begin{array}{lcl} \dfrac{\sin(x)}{x}&\text{ si }&x\neq 0\\ 1&\text{ si }& x=0 \end{array} \right.$. Alors l'ensemble des solutions DSE de $(E)$ est $\Vect (f)$.

($\star\star$) Correction

On pose, pour $x\in \R$, $J(x)=\dsp\frac{1}{\pi}\int_0^\pi \cos(x\sin(\theta))\ud \theta$.

  1. Montrer que $J\in \CC^\infty(\R)$. Calculer $J^{(n)}$. Montrer que $J$ est DSE puis déterminer son développement en série entière.
  2. Montrer que $J$ est une solution de l'équation différentielle $xy''+y'+xy=0$.

Correction

La fonction $\fonct{\varphi}{\R\times [0,\pi]}{\R}{(x,t)}{\cos(x\sin(t))}$ est de classe $\CC^\infty$, d'après le théorème de cours sur les intégrales à paramètre (dans la cas ou $I$ est un segment), $J$ est de classe $\CC^\infty$ sur $\R$, de plus, pour tout $n\in \N$, on a $$\forall x\in \R,\,J^{(n)}(x)=\frac{1}{\pi}\int_0^\pi \dfrac{\partial^n \varphi}{\partial x^n}(x,t)\ud t=\dfrac{1}{\pi }\int_0^\pi \sin(t)^n\cos\left(x\sin(t)+\frac{n\pi}{2}\right)\ud t.$$ D'autre part, pour tout $(x,t)\in \R\times [0,\pi]$, on a $\cos(x\sin(t))=\dsum_{n\geq 0}\dfrac{(-1)^n\sin(t)^{2n}x^{2n}}{(2n)!}$, on pose alors pour $x\in \R$ et $n\in \N$ $\fonct{u_n}{\R}{\R}{t}{\dfrac{(-1)^n\sin(t)^{2n}x^{2n}}{(2n)!}}$ alors $\textstyle\sum\limits u_n$ converge normalement sur $\R$ puisque $\norme{u_n}_\infty\leq \dfrac{x^{2n}}{(2n)!}$ et $\textstyle\sum\limits \dfrac{x^{2n}}{(2n)!}$ converge, on en déduit qu'on peut inverser l'intégrale et la somme: $$\begin{array} {lcl} \forall x\in \R,\,\,J(x)&=&\dsp\dfrac{1}{\pi}\int_0^\pi \cos(x\sin(t))\ud t = \dfrac{1}{\pi}\int_0^\pi \sum\limits_{n\geq 0}\dfrac{(-1)^n\sin(t)^{2n}x^{2n}}{(2n)!}\ud t\\ &&\\ &=&\dsp\dfrac{1}{\pi}\sum\limits_{n\geq 0} \int_0^\pi \dfrac{(-1)^n\sin(t)^{2n}x^{2n}}{(2n)!}\ud t =\textstyle\sum\limits_{n\geq 0} \left(\int_0^\pi \dfrac{(-1)^n\sin^{2n}(t)}{\pi (2n)!}\ud t\right) x^{2n}. \end{array} $$ On reconnait l'intégrale de Wallis (voir corrigé de DS 2), on en déduit, $$\forall x\in \R,\, J(x)=\dsum_{n\geq 0} \dfrac{(-1)^n}{4^n(n!)^2}x^{2n}.$$ Remarque: Un calcul simple montre que le rayon de convergence de la série $\dsum \dfrac{(-1)^n}{4^n(n!)^2}x^{2n}$ est $\infty$.
On peut montrer que $J$ est la solution de $(E)$ en utilisant son DSE, i.e. pour tout $x\in \R$, $$\begin{array} {lcl} xJ''(x)+J'(x)+xJ(x)&=&\dsum_{n\geq 1}\dfrac{(-1)^n(2n)(2n-1)}{4^n(n!)^2}x^{2n-1}+\dsum_{n\geq 1}\dfrac{(-1)^n(2n)}{4^n(n!)^2}x^{2n-1}+\dsum_{n\geq 0}\dfrac{(-1)^n}{4^n(n!)^2}x^{2n+1}\\ &&\\ &=&\dsum_{n\geq 0}\dfrac{(-1)^{n+1}(2n+2)(2n+1)}{4^{n+1}((n+1)!)^2}x^{2n+1}+ \dsum_{n\geq 0}\dfrac{(-1)^{n+1}(2n+2)}{4^{n+1}((n+1)!)^2}x^{2n+1}\\ &&+\dsum_{n\geq 0}\dfrac{(-1)^{n}}{4^{n}(n!)^2}x^{2n+1}\\ &&\\ &=&\dsum_{n\geq 0}\dfrac{(-1)^{n+1}}{4^{n}(n!)^2}\left(\dfrac{2n+1}{2(n+1)}+\dfrac{1}{2(n+1)}-1\right) x^{2n}\\ &&\\ &=&\dsum_{n\geq 0}\dfrac{(-1)^{n+1}}{4^{n}(n!)^2}\left(\dfrac{2n+1}{2(n+1)}+\dfrac{1}{2(n+1)}-\dfrac{2n+2}{2(n+1)}\right) x^{2n}=0 \end{array} $$ Donc $J$ est une solution de l'équation $(E)$.
On aurait pu également utiliser la définition de $J$, $$\begin{array} {lcl} xJ''(x)+J'(x)+xJ(x)&=&\dsp\dfrac{1}{\pi}\int_0^\pi \left(-x\sin^2(t)\cos(x \sin(t))-\sin(t)\sin(x\sin(t))+x\cos(x\sin(t))\right)\ud t\\ &&\\ &=&\dsp\dfrac{1}{\pi}\int_0^\pi \left(x\cos^2(t)\cos(x\sin(t))-\sin(t)\cos(x\sin(t))\right)\ud t\\ &&\\ &=& \dsp\dfrac{1}{\pi}\int_0^\pi \left(\cos(t)\sin(x\sin(t))\right)'\ud t=\left[\cos(t)\sin(x\sin(t))\right]_0^\pi =0 \end{array} $$ Ce qui montre que $J$ est une solution de l'équation $(E)$.

($\star\star$) Correction

Résoudre sur $\R_+^*$ l'équation différentielle $2xy'+y=3x\cos( x^{3/2})$.

Correction

L'ensemble de solutions de l'équation homogène est $S_H=\{x \in \R_+^*\longmapsto \dfrac{a}{\sqrt{x}},\,\,a\in \R\}$.
On va chercher une solution particulière sous la forme d'une somme d'une série entière, puisque sur $\R_+$ on peut écrire $3x\cos \left(x^{3/2}\right)=\dsum_{n\geq 0}(-1)^n\dfrac{3x^{3n+1}}{(2n)!}$.
Soit $f(x)=\dsum_{n\geq 0}a_nx^n$ la somme d'une série entière de rayon de convergence $R\geq 0$ telle que $f_{\mid_{\R_+^*}}$ soit une solution de $E$. Alors, $$\forall x\in ]0,R[,\quad \dsum_{n\geq 0}(2n+1)a_nx^n=\dsum_{n\geq 0}(-1)^n\dfrac{3x^{3n+1}}{(2n)!}$$ soit $$ \forall n\in \N,\,a_n=\left\{\begin{array}{lcl} 0&\text{ si }n\neq 1[3]\\ &&\\ \dfrac{3(-1)^k}{(6k+3) (2k)!}&\text{ si } n=3k+1 \end{array} \right.$$ Donc $f(x)=\dsum_{n\geq 0}(-1)^n\dfrac{x^{3n+1}}{(2n+1)!}$, un calcul simple montre que $R=\infty$. D'autre part, on a $$\forall x>0,\,\,f(x)=\dsum_{n\geq 0}(-1)^n\dfrac{x^{3n+1}}{(2n+1)!}=f(x)=\dsum_{n\geq 0}(-1)^n\dfrac{\left(x^{3/2}\right)^{3n+2}}{\sqrt{x}(2n+1)!}=\dfrac{\sin \left(x^{3/2}\right)}{\sqrt{x}}.$$ On en déduit que l'ensemble de solutions de l'équation est $S=\left\{x\in \R_+^*\longmapsto \dfrac{\sin \left(x^{3/2}\right)+a}{\sqrt{x}},\,\,a\in \R\right\}$.

(CCP PC 2001) Correction

Résoudre $\,4xy''-2y'+9x^2y=0$ en cherchant une solution développable en série entière.

Correction

Soit $f(x)=\dsum_{n\geq 0}a_nx^n$ une somme de SE de rayon de convergence $R\geq 0$, supposons que $f$ est une solution de $E$, alors $\forall x\in ]-R,R[,\,f'(x)=\dsum_{n\geq 1}na_nx^{n-1},\,f''(x)=\dsum_{n\geq 2}n(n-1)a_nx^{n-2}$ ce qui donne $$\begin{array} {lcl} \forall x\in ]-R,R[,\,\,4xf''(x)-2f'(x)+9x^2f(x)&=&\dsum_{n\geq 1}2n\left(2n-3)\right)a_nx^{n-1}+ \dsum_{n\geq 0}9a_nx^{n+2}\\ &= &\dsum_{n\geq 0}2\left(n+1\right)(2n-1)a_{n+1}x^{n}+\dsum_{n\geq 2}a_{n-2}x^{n} \end{array} $$ Donc $f$ est une solution ssi pour tout $x\in ]-R,R[$ $$0=4a_1+4a_2x+\dsum_{n\geq 2}\left(2(n+1)(2n-1)a_{n+1}+9a_{n-2}\right)x^{n}\Longrightarrow\left\{\begin{array}{l} a_1=a_2=0\\ \forall n\geq 1,\,\, a_{n+1}=\dfrac{-9a_{n-2}}{2(n+1)(2n-1)}. \end{array}\right. $$ On trouve une relation de récurrence $2n(2n-3)\,a_n = -9a_{n-3}$ et donc des coefficients $\displaystyle a_{3p}=\dfrac{(-1)^p}{(2p)!}a_0$. C'est-à-dire qu'une solution est $$f(x)=\begin{cases}a_0\,\cos(x^{3/2}) & \text{si }x\geq 0,\\ &&\\ a_0\ch(x^{3/2}) & \text{si }x\leq 0. \end{cases} $$

($\star$) Correction

Soit $E = \CC^\infty(\R,\C)$ et $\Phi : E \to E$, définie par $\Phi(f) = g$ où $g$ est l'application ${g:t \mapsto f'(t)+tf(t).}$

  1. Trouver les valeurs propres et les vecteurs propres de $\Phi$.
  2. Trouver les valeurs propres et les vecteurs propres de $\Phi^2$.
  3. Résoudre l'équation : $y'' + 2xy' + (x^2-1)y = 0$.

Correction

  1. spectre $\Sp (\phi)= \C$, $f_\lambda(t) = e^{-t^2/2}e^{\lambda t}$.
  2. Pour $\lambda \ne 0$, $\Phi^2(f) = \lambda^2 f \Leftrightarrow f = af_\lambda + bf_{-\lambda}$. Pour $\lambda = 0$, $\Phi^2(f) = 0 \Leftrightarrow f(t) = (at+b)e^{-t^2/2}$.
  3. $\Phi^2(y)=-2y \Leftrightarrow y = e^{-t^2/2}\bigl( a\cos(t\sqrt2\,) + b\sin(t\sqrt2\,)\bigr)$.

(CCP PSI 2019) Correction

Soit $(u_n)$ la suite définie par $u_0=3$ et pour tout $n\geq 0,\,u_{n+1}=\dsum_{k=0}^n\binom{n}{k}u_ku_{n-k}$.

  1. Montrer que $\forall n\in \N,\ \dfrac {u_n}{n!}\leqslant 4^{n+1}$.
  2. Montrer que la fonction somme $S:x\mapsto S(x)=\displaystyle\sum_{n=0}^{+\infty}\frac {u_n}{n!}x^n$ est solution de l'équation différentielle $y'=y^2$ sur un intervalle que l'on précisera.
  3. En déduire une expression de $u_n$ pour tout $n\in\N$.

Correction

  1. Par récurrence forte sur $n\in \N$.
  2. D'après la question précédente, le rayon de convergence $R$ de $S$ vérifie $R\geq 1/4>0$.
    Pour tout $x\in ]-R,R[$, on a $$ \begin{array}{lcl} S'(x)&=& \dsum_{n\geq 1}\dfrac{u_n}{(n-1)!}x^{n-1}= \dsum_{n\geq 0}\dfrac{u_{n+1}}{n!}x^{n}\\ &&\\ &=& \dsum_{n\geq 0}\left(\dsum_{k=0}^n\dfrac{u_ku_{n-k}}{k!(n-k)!}\right)x^n=S(x)^2. \end{array} $$ Ainsi, $S$ est la solution du problème de Cauchy $$ \forall x\in ]-R,R[,\quad y'=y^2 ,\quad y(0) =3.$$ La résolution de ce problème donne : $$\forall x\in ]-R,R[,\quad S(x) = \dfrac{3}{1-3x}.$$ On déduit également que $R =1/3$.
  3. Pour $x\in ]-1/3,1/3[$, on a $$ \dfrac{3}{1-3x} =3 \dsum_{n\geq 0} (3x)^n =\dsum_{n\geq 0} 3^{n+1}x^n.$$ On en déduit alors: $$\forall n\in \N,\quad u_n = n! (3^{n+1}).$$
(TPE-EIVP PSI 2018) Correction

Soit $(E):y''-(2+4x^2)y=0$. Soit $f$ une solution de $(E)$ telle que $f(0)=1$. On suppose qu'il existe $R>0$ et $(a_n)\in\R^\N$ tels que $a_0=1$ et $\forall x\in\left]-R,R\right[,\, f(x)=\dsum_{n=0}^{+\infty}a_nx^{2n}$.

  1. Déterminer $a_1$.
  2. Déterminer une relation entre $a_{n+1}$, $a_n$ et $a_{n-1}$.
  3. Déterminer $a_2$ et $a_3$. Faire une conjecture sur $a_n$ et la démontrer.
  4. Résoudre $(E)$ en posant $y(x)=f(x)z(x)$.

Correction

  1. $a_1=f''(0)/2$ or $f''(0)=2f(0)=2$ donc $a_1=1$.
  2. Pour tout $x\in ]-R,R[$, on a: $$f'(x)=\dsum_{n\geq 1}2na_nx^{2n-1},\quad f''(x)=\dsum_{n\geq 1}(2n)(2n-1)a_nx^{2n-2}=\dsum_{n\geq 0}(2n+2)(2n+1)a_{n+1}x^{2n}.$$ En remplaçant dans l'équation différentielle, on trouve pour tout $x\in ]-R,R[$, $$ \begin{array}{lcl} f''-(2+4x^2)f&=& \dsum_{n\geq 0}(2n+2)(2n+1)a_{n+1}x^{2n}-2\dsum_{n\geq 0}a_{n}x^{2n}-4\dsum_{n\geq 0}a_{n}x^{2n+2}\\ &&\\ &=& \dsum_{n\geq 0}((2n+2)(2n+1)a_{n+1}-2a_n)x^{2n}-4 \dsum_{n\geq 1}a_{n-1}x^{2n}\\ &&\\ &=& 2a_1-2a_0 + \dsum_{n\geq 1} \left( (2n+2)(2n+1)a_{n+1}-2a_n-4a_{n-1}\right)x^n =0 \end{array} $$ Ainsi, $$\forall n\geq 1,\quad (2n+2)(2n+1)a_{n+1}-2a_n-4a_{n-1}=0$$
  3. En utilisant la question précédente, on trouve : $$ a_2 =\dfrac{6}{12}=\dfrac{1}{2},\quad a_3 =\dfrac{2\times \frac12 +4}{6\times 5} =\frac16.$$ Puis par récurrence on montre que $a_n =\dfrac{1}{n!}$, ce qui donne $$f(x)=\dsum_{n\geq 0} \dfrac{x^{2n}}{n!} =\exp (x^2),\quad (\text{donc } R=\infty).$$
  4. Soit $y$ une solution de $(E)$, on pose $z=y/f$ donc $z\in \CC^2(\R)$ ($f$ ne s'annule pas sur $\R$), ainsi, $$ y'= f' z+fz',\, y''=f''z +2f'z' +fz''$$ Puis, $$0=y''-(2+4x^2) y = 2f'z'+fz'' = (4xz'+z'') \exp(x^2)$$ donc $z'(x) =\alpha \exp(-2x^2)$ ($\alpha\in \R$), soit $z(x) = z(0)+\alpha \dsp\int_0^x\exp(-2t^2)\ud t$. Donc $$ y(x) = a \exp (x^2)+b\exp(x^2) \int_0^x\exp(-2t^2)\ud t.$$
(Mines-Ponts PSI 2019) Correction

Soit $f$ définie par $f(t)=\cos\left(\Arcsin(t)/2\right)$.

  1. Donner le domaine de définition de $f$ et donner une équation différentielle d'ordre 2 vérifiée par $f$.
  2. En déduire un développement en série entière de $f$.

Correction

  1. $f$ est définie sur $[-1,1]$, et vérifie: $$(1-x^2)y''-xy'+\frac{1}{4}y=0.$$
  2. $f=\dsum a_{2n}x^{2n}$ avec $a_{2n}=\dfrac{2 (2n-2)!}{4^n\,n!(n-1)!}$.

(CCINP PSI 2019) Correction

On considère la suite $(a_n)$ définie par: $a_0=a_1=1$ et pour tout $n\in \N,\, a_{n+2}=a_{n+1}+(n+1)a_n$.

  1. Montrer que : $\forall n\in \N,\, a_n\leq n!$.
  2. Qu'en déduit-on sur le rayon de convergence de la série entière $\dsum \dfrac{a_n}{n!}x^n$, on note $f$ sa somme.
  3. Trouver une équation différentielle d'ordre 1 dont $f$ est solution et la résoudre.
  4. Pour $p\in \N$, calculer $a_{2p}$ et $a_{2p+1}$ en fonction de $p$.

Correction

  1. Par récurrence sur $n\in \N$.
  2. D'après la question précédente, $a_n\leq n!$ donc $\dfrac{a_n}{n!}\leq 1$, ce qui implique que $R$ (le rayon de convergence de la série $\dsum \dfrac{a_n}{n!}x^n$) est supérieure ou égale à $1$.
  3. Soit $x\in ]-R,R[$, on a $$ \begin{array}{lcl} f'(x)&=&\dsum_{n\geq 1} \dfrac{a_n}{n!}nx^{n-1} = \dsum_{n\geq 1} \dfrac{a_n}{(n-1)!}x^{n-1}\\ &&\\ &=&\dsum_{n\geq 0} \dfrac{a_{n+1}}{n!}x^{n} = a_1 + \dsum_{n\geq 1} \dfrac{a_{n+1}}{n!}x^{n}\\ &&\\ &=& 1 + \dsum_{n\geq 0} \dfrac{a_{n+2}}{(n+1)!}x^{n+1}\\ &&\\ &=& 1 + \dsum_{n\geq 0} \left( \dfrac{a_{n+1}}{(n+1)!}+\dfrac{a_n}{n!}\right) x^{n+1}\\ &&\\ &=&1+\dsum_{n\geq 1} \dfrac{a_n}{n!} x^{n} + x\dsum_{n\geq 0} \dfrac{a_n}{n!} x^{n} = f(x)+xf(x) \end{array} $$ Donc $f$ est une solution de l'équation différentielle $y'-(1+x)y=0$ avec $f(0)=1$. La résolution de cette équation donne: $$\forall x\in ]-R,R[,\quad f(x) = \exp (x + \frac{x^2}{2}).$$
  4. En utilisant le produit de Cauchy de $\exp(x)$ avec $\exp(x^2/2)$, on trouve $$\forall n\in \N,\quad a_{2n}=(2n)!\dsum_{j=0}^n \dfrac{1}{(2n-2j)!}\dfrac{1}{j!}\dfrac{1}{2^j}.$$ $$\forall n\in \N,\quad a_{2n+1}=(2n+1)!\dsum_{j=0}^n \dfrac{1}{(2n+1-2j)!}\dfrac{1}{j!}\dfrac{1}{2^j}.$$
(Mines-Télécom PSI 2019) Correction

On considère le problème de Cauchy $\left\{\begin{array}{l} y''+2xy'+2y=0,\\ y(0)=1,\,\,y'(0)=0. \end{array} \right.$.
Déterminer l'unique solution sous forme de somme d'une série entière. L'exprimer à l'aide des fonctions usuelles.

Correction

Soit $f(x)=\dsum_{n\geq 0}a_nx^n$ une somme de SE de rayon de convergence $R\geq 0$, supposons que $f$ est une solution de $E$, alors $\forall x\in ]-R,R[,\,f'(x)=\dsum_{n\geq 1}na_n x^{n-1},\, f''(x)=\dsum_{n\geq 2}n(n-1)a_nx^{n-2}$ ce qui donne, pour tout $x\in ]-R,R[$ $$\begin{array} {lcl} f''(x)+2xf'(x)+2f(x)&=&\dsum_{n\geq 2}n(n-1)a_nx^{n-2}+2\dsum_{n\geq 0}na_n x^{n}+2\dsum_{n\geq 0}a_nx^{n}\\ &=& \dsum_{n\geq 0}(n+2)(n+1)a_{n+2}x^{n}+\dsum_{n\geq 0}2(n+1)a_{n}x^{n} \end{array} $$ Donc $f$ est une solution ssi pour tout $x\in ]-R,R[$ $$0=\dsum_{n\geq 0}\left((n+2)(n+1)a_{n+2}+2(n+1)a_{n}\right)x^{n}$$ Ce qui implique $$ a_0=1,\,a_1=0,\, \,\forall n\geq 0,\,\, a_{n+2}=-\dfrac{2a_n}{(n+2)}. $$ Ceci nous donne, pour tout $k\in \N$, $$\begin{array} {lcl} a_{2k+1}&=&0,\,\,\\ a_{2k+2}&=&\dfrac{-2 a_{2k}}{(2k+2)}=\dfrac{4a_{2k-2}}{(2k+2)(2k)}=\cdots=\dfrac{(-1)^ka_0}{(k+1)!}. \end{array} $$ On en déduit alors que $f(x)=\dsum_{n\geq 0}\dfrac{(-1)^nx^{2n}}{(n)!}=\ee^{-x^2}$.

(CCINP PSI 2019) Correction

On note pour $n\in \mathbb{N},\ \ a_n=\displaystyle \int_0^1 \left(\frac{1+t^2}{2}\right)^n\,\ud t$.

  1. Montrer que la suite $(a_n)_{n\in \mathbb{N}}$ est bien définie et déterminer sa limite.
  2. Étudier la convergence de la série de terme général $(-1)^n\, a_n$.
  3. Soit la série entière $\dsum a_nx^n$. On note $f$ sa somme. Montrer que : $\ \ \forall n\in \mathbb{N},\quad a_n\geqslant \frac{1}{2n+1} \quad$, calculer le rayon de convergence , puis montrer que $f$ est solution d'une équation différentielle.

Correction

  1. Pour $n\in \N$, $t\longmapsto \left(\frac{1+t^2}{2}\right)^n$ est continue sur $[0,1]$ dons $a_n$ est bien définie. De plus, pour tout $t\in [0,1]$, on a $$0\leq \dfrac{1+t^2}{2}\leq \dfrac{1+t}{2}\Longrightarrow 0\leq a_n \leq \dfrac{2}{n+1}$$ Ce qui implique $a_n\tendversN\, 0$.
  2. Pour $t\in [0,1]$, on a $\dfrac{1+t^2}{2}\in [0,1]$ donc $$\forall n\in \N,\, \left(\dfrac{1+t^2}{2}\right)^{n+1}\leq \left(\dfrac{1+t^2}{2}\right)^{n}\Longrightarrow a_{n+1}\leq a_n$$ Donc la suite $(a_n)$ est décroissante.
    On en déduit alors que $\dsum (-1)^na_n$ converge en utilisant CSSA.
  3. Soit $n\geq 1$, on a $$\forall t\in [0,1],\, t^2\leq t\leq \dfrac{1+t^2}{2}\Longrightarrow \dfrac{1}{2n+1}= \int_0^1t^{2n}\ud t\leq a_n$$ Comme le rayon de convergence de la SE $\sum \dfrac{x^n}{2n+1}$ égale à $1$, on en déduit que le rayon de convergence de $\sum a_n x^n$ est $\leq 1$, mais vu que $\dsum (-1)^n a_n$ converge alors $R_c=1$.
    D'autre part, en faisant une IPP on obtient la relation suivante: $$a_0= 1,\, \forall n\geq 1,\, a_{n}=\dfrac{1+na_{n-1}}{2n+1}.$$ En effet, pour $n\geq 1$, on a : $$\begin{array}{lcl} \dsp\int_0^1\left(\dfrac{1+t^2}{2}\right)^n\ud t &=& \dsp \left[t\left(\dfrac{1+t^2}{2}\right)^n\right]_0^1 -n \int_0^1t^2\left(\dfrac{1+t^2}{2}\right)^{n-1}\ud t\\ &&\\ &=&\dsp 1-n\int_0^12\left(\frac{1+t^2-1}{2}\right)\left(\dfrac{1+t^2}{2}\right)^{n-1}\ud t\\ &&\\ &=& 1- 2n a_n +na_{n-1} \end{array} $$ Pour tout $x\in ]-1,1[$, on a $$\begin{array} {lcl} 2xf'(x)&=& \dsum_{n\geq 1}2na_nx^n = \dsum_{n\geq 1}(2n+1-1)a_nx^n\\ &&\\ &=&\dsum_{n\geq 1}(1+na_{n-1})x^n-\dsum_{n\geq 1} a_nx^n\\ &&\\ &=& \dsum_{n\geq 1}x^n +\dsum_{n\geq 1}(n-1)a_{n-1}x^n+\dsum_{n\geq 1}a_{n-1}x^n-f(x)+1\\ &&\\ &=&\dfrac{1}{1-x}+x^2f'(x)+xf(x)-f(x) \end{array} $$ On en déduit que $f$ vérifie l'équation différentielle suivante: $$\forall x\in ]-1,1[,\, \, x(2-x)f'(x)+(1-x)f(x)=\dfrac{1}{1-x},\quad \text{ et } f(0)=1.$$

(CCINP PC 2019) Correction

On pose $S(x) = \displaystyle \sum_{n=0}^{+\infty} \binom{2 n}{n} x^n$.

  1. Rayon de convergence ? On le note $R$.
  2. Pour tout $x$ dans $]-R, R[$, prouver la relation $(1 - 4x)S'(x) - 2 S(x) = 0$. En déduire une expression de $S(x)$.

Correction

  1. On note $u_n=\binom{2 n}{n}=\dfrac{(2n)!}{(n!)^2}$, en utilisant la regle de D'Alambert, on trouve $$\dfrac{u_{n+1}}{u_n}=\dfrac{(2n+2)!}{((n+1)!)^2}\dfrac{(n!)^2}{(2n)!}=\dfrac{(2n+2)(2n+1)}{(n+1)^2}\tendversN\, 4.$$ On en déduit que le rayon de convergence de $\dsum u_n x^n$ est $\dfrac14$.
  2. Pour tout $x\in ]-1/4,1/4[$, on a $$\begin{array} {lcl} (1 - 4x)S'(x) - 2 S(x) &=& \dsum_{n\geq 1}nu_nx^{n-1}-4\dsum_{n\geq 0}nu_nx^n-2\dsum_{n\geq 0}u_nx^n\\ &=&\dsum_{n\geq 0}(n+1)u_{n+1}x^{n}-2\dsum_{n\geq 0}(2n+1)u_nx^n\\ &=&\dsum_{n\geq 0}\left((n+1)u_{n+1}-2(2n+1)u_n\right)x^n=0 \end{array} $$ puisque, pour tout $n\geq 0,\, u_{n+1}=\dfrac{2(2n+1)}{n+1}u_n$.
    La résolution de cette équation différentielle (avec la condition $S(0)=1$), donne $$\forall x\in ]\frac{-1}{4},\frac14[,\quad S(x)=\dfrac{1}{\sqrt{1-4x}}.$$

Divers

($\star\star$) HP Correction

Montrer l'existence et l'unicité d'une solution bornée sur $\R$ de l'équation différentielle $(E)\,\, y'' - y = \ee^{-x^2}$. Expliciter cette solution.

Correction

Les solutions de l'équation homogène sont les fonctions définies sur $\R$ de la forme $f(x)=a\ee^x+b^{-x},\, (a,b\in \R)$.
On va utiliser la méthode des variations des constante pour trouver une solution particulière de $(E)$. On cherche alors $a,b:\R\longmapsto \R$ de classe $\CC^1$ telles que: $$\forall x\in \R,\, \left\{\begin{array}{lcl} a'(x)\ee^x+b'(x)\ee^{-x}&=&0\\ a'(x)\ee^x-b'(x)\ee^{-x}&=&\ee^{-x^2} \end{array} \right. $$ soit $$\forall x\in \R,\quad a(x)=\dfrac{1}{2}\int_0^x\ee^{-t^2-t}\ud t,\,\,b(x)=\dfrac{-1}{2}\int_0^x\ee^{-t^2+t}\ud t .$$ Les solutions de l'équation $(E)$ sont les fonction définie sur $\R$ de la forme: $$\forall x\in \R,\quad y(x)=\ee^x\left( a+\dfrac{1}{2}\int_0^x\ee^{-t^2-t}\ud t\right)+\ee^{-x}\left( b-\dfrac{1}{2}\int_0^x\ee^{-t^2+t}\ud t\right),\quad a,b\in \R.$$ Les intégrales $\dsp\int_{-\infty}^\infty \ee^{-t^2-t}\ud t$ (resp. $\dsp\int_{-\infty}^\infty \ee^{-t^2+t}\ud t$) sont convergentes.
Pour qu'une solution $f$ de $(E)$ soit borné sur $\R$ il faut que (condition nécessaire): $a=-\dfrac{1}{2}\dsp \int_0^\infty \ee^{-t^2-t}\ud t$ (sinon $\dsp\limiteX{x}{\infty}f(x)=\infty$) et $b=\frac{-1}{2}\dsp\int_{-\infty}^0 \ee^{-t^2+t}\ud t$.
Donc le seul candidat est la fonction: $$\begin{array} {lcl} f(x)&=&\dsp\dfrac{\ee^x}{2}\left(\int_0^x \ee^{-t^2-t}\ud t-\int_0^\infty \ee^{-t^2-t}\ud t\right)-\dfrac{\ee^{-x}}{2}\left(\int_0^x \ee^{-t^2+t}\ud t+\int_{-\infty}^0 \ee^{-t^2+t}\ud t\right)\\ &&\\ &=&\dsp\dfrac{-\ee^x}{2}\int_x^\infty\ee^{-t^2-t}\ud t-\dfrac{\ee^{-x}}{2}\int_{-\infty}^x\ee^{-t^2+t}\ud t . \end{array} $$ Montrons que cette solution est bornée sur $\R$.

  1. pour tout $t>1$ on a $\ee^{-t^2-t}\leq \ee^{-3t/2}$ et $\dsp\int_1^\infty\ee^{-3t/2}\ud $ converge. Donc, pour tout $x>1$, $$0< \ee^{x}\int_x^\infty \ee^{-t^2-t}\ud t\leq \ee^x\int_x^\infty\ee^{-3t/2}\ud t=\dfrac{2\ee^{-x/2}}{3}\Longrightarrow\limiteX{x}{\infty} \dfrac{-\ee^x}{2}\int_x^\infty\ee^{-t^2-t}\ud t=0.$$ D'autre par, $\limiteX{x}{\infty} \dsp\dfrac{\ee^{-x}}{2}\int_{-\infty}^x\ee^{-t^2-t}\ud t=0$. On en déduit que $f(x)\tendvers{x}{\infty}\,0$.
  2. Soit $x\in \R$, on a $$\begin{array} {lcl} f(-x)& =&\dsp \dfrac{-\ee^{-x}}{2}\int_{-x}^\infty\ee^{-t^2-t}\ud t-\dfrac{\ee^{x}}{2}\int_{-\infty}^{-x} \ee^{-t^2+t}\ud t \\ &&\\ &=&\dsp\dfrac{-\ee^{-x}}{2}\underset{u=-t}{\int_{-\infty}^x}\ee^{-u^2+u}\ud u-\dfrac{\ee^{x}}{2}\underset{u=-t} {\int_x^{\infty}}\ee^{-t^2-t}\ud t=f(x) \end{array} $$ Donc $f$ est paire d'où $f(x)\tendvers{x}{-\infty}\,0$.

On en déduit que $f$ est bornée sur $\R$.

($\star\star$) HP Correction

Résoudre sur $]0,\pi[$ l'équation différentielle: $y''+y=\dfrac{1}{\sin(x)}$.

Correction

Les fonction $\cos, \,\sin$ forment un système fondamentale de solutions de l'équation homogène.
On va utiliser la méthode des variations des constantes pour chercher une solution particulière. On cherche alors $y$ sous la forme $\lambda \cos +\beta \sin $ ou $\lambda,\,\beta\in \CC^1(]0,\pi[,\R)$ telles que: $$ \forall x\in ]0,\pi[,\quad \left\{\begin{array}{lcl} \lambda '(x)\cos(x)+\beta'(x)\sin(x)&=& 0\\ -\lambda'(x)\sin(x)+\beta'(x)\cos(x)&=&\dfrac{1}{\sin(x)} \end{array} \right.$$ On en déduit que $\beta'=\dfrac{\cos}{\sin},\,\,\lambda'=-1$. Ce qui donne $$\forall x\in ]0,\pi[,\quad \lambda(x)=-x+a,\quad\beta(x)=\ln(\sin(x))+b,\quad a,b\in \R.$$ Donc l'ensemble de solutions $S=\{x\longmapsto a\cos(x)+b\sin(x)-x\cos(x)+\ln(\sin(x))\sin(x),\,a,b\in \R\}$.

($\star$) Correction

Soit $f$ une application continue par morceaux de $\R$ dans~$\R$ admettant une limite finie en $+\infty$.
Montrer que toute solution de l'équation différentielle $y'+y=f(x)$ admet une limite finie en $+\infty$.

Correction

On connaît les solutions de l'équation homogène : $y(x)=\lambda\ee^{-x}$ ; on trouve donc une solution particulière par la méthode de variation de la constante et au final, l'ensemble de solutions est: \begin{equation*} \mathcal{S}=\left\{ x\longmapsto \ee^{-x}\left[\dsp\int_0^x \ee^{t}\,f(t)\,\ud t+\lambda\right], \,\,\lambda\in \R\right\}. \end{equation*} Supposons que $\lim f=0$, et posons $h(x)=\ee^{-x}\dsp\int_0^x\ee^{t}\,f(t)\,\ud t$. Alors $f$ est bornée sur $\R_+$ : $\exists A>0,~~\forall x\in \R_+,~~\abs{f(x)}\leq A$.
Soit $\varepsilon>0$, il existe $X\geq 0$ tel que pour tout $t\geq X$, on a $\abs{f(t)}\leq\varepsilon$. Alors \begin{equation*} \forall x\geq 2X,\qquad \abs{h(x)}\leq A\dsp\int_0^{x/2}\ee^{t-x}\,\ud t + \varepsilon\dsp\int_{x/2}^x \ee^{t-x}\,\ud t=A\ee^{-x/2}+\varepsilon, \end{equation*} qui est plus petit que $2\varepsilon$ pour $x$ suffisamment grand. Donc $\displaystyle \lim_{x\to+\infty}h(x)=0$.
Dans le cas général, si $f$ a pour limite $\ell$, on considère $f-\ell$ et on montre que $h$ a pour limite $\ell$.

(CCP PSI) Correction

On considère l'équation $y'+y=f(x)$, où $f$ est une fonction continue.

  1. Trouver toutes les solutions de cette équation.
  2. On suppose de plus que $f$ est $T$-périodique. Peut-on trouver une solution $T$-périodique ?

Correction

  1. $\mathcal{S}=\left\{x\longmapsto\dsp\int_0^x f(t)\,\ee^{t-x}\,\ud t + \lambda\,\ee^{-x},\,\,\lambda\in \R\right\}$.
  2. Une condition nécessaire pour que $y$ soit $T$-périodique est que $y(T)=y(0)$, ce qui impose immédiatement la condition nécessaire \begin{equation*} y(0)=y(T)\Longrightarrow \lambda=\dsp\int_0^T \ee^{t-T}\,f(t)\,\ud t + \lambda\,\ee^{-T} \end{equation*} et donc \begin{equation*} \lambda = \dfrac{\dsp\int_0^T f(t)\,\ee^{t-T}\,\ud t}{1-\ee^{-T}}. \end{equation*} On montre maintenant que c'est une condition suffisante :
    Avec la valeur trouvée de $\lambda$, on a, pour tout $x\in\R$ : $$\begin{array} {lcl} y(x+T)-y(x)&=&\ee^{-(x+T)}\dsp\int_0^{x+T} \ee^{t}\,f(t)\,\ud t - \ee^{-x} \dsp\int_0^x \ee^{t}\,f(t)\,\ud t +\lambda (\ee^{-T}-1)\,\ee^{-x} \\ &=& \ee^{-(x+T)}\left(\dsp\int_0^T\ee^{u}\,f(u)\,\ud u + \dsp\int_T^{x+T}\ee^{u}\,f(u)\,\ud u\right)\\ && -\ee^{-x}\dsp\int_0^x \ee^{t}\,f(t)\,\ud t - \ee^{-(x+T)}\dsp\int_0^T \ee^{t}\,f(t)\,\ud t\\ &=& \ee^{-(x+T)}\dsp\int_0^x \ee^{T+t}\,\underbrace{f(T+t)}_{=f(t)}\,\ud t - \ee^{-x} \dsp\int_0^x \ee^t\,f(t)\,\ud t\\ &=&0 \end{array} $$

($\star\star$) HP Correction

Soit $f:\R\longrightarrow\R$ de classe~$\CC^2$ telle que \begin{equation*} \forall x\in\R\qquad f(x)+f''(x)\geq 0. \end{equation*} Montrer que $f(x)+f(x+\pi)\geq 0$ pour tout $x\in\R$. (On pourra poser $g=f+f''$.)

Correction

Il existe une fonction continue $g:\R\longrightarrow\R^+$ telle que \begin{equation*} y''+y=g. \end{equation*} On utilise alors la méthode de variations des constantes : il existe $u,v$ de classe~$\CC^1$ telles que \begin{equation*} \left\{ \begin{aligned} f(x) & = u(x)\cos x +v(x)\sin x, \\ f'(x) &= -u(x)\sin x +v(x)\cos x. \end{aligned} \right. \end{equation*} et $(u',v')$ vérifie le système \begin{equation*} \left\{ \begin{aligned} u'(x)\cos x + v'(x)\sin x & = 0,\\ -u'(x) \sin x+ v'(x)\cos x & =g(x). \end{aligned} \right. \end{equation*} On en déduit $u'(x)=-g(x)\sin x$ et $v'(x)=g(x)\cos x$, et donc \begin{equation*} u(x)=a-\dsp\int_0^x g(t)\sin t \,\ud t ,\qquad v(x)=b+\dsp\int_0^x g(t)\cos t\,\ud t. \end{equation*} Enfin, on obtient \begin{equation*} \boxed{f(x)=a\cos x +b\sin x +\dsp\int_0^x g(t)\sin(x-t)\,\ud t}. \end{equation*} On remarque alors que $$ \begin{array}{lc} f(x)+f(x+\pi) & =\dsp\int_0^x g(t)\sin(x-t)\,\ud t + \dsp\int_0^{x+\pi} g(t)\sin(x+\pi-t)\,\ud t \\ & = \dsp\int_{x+\pi}^{x}g(t)\sin(x-t)\,\ud t = \dsp\int_0^\pi g(x-u)\sin u\,\ud u\geq 0 \end{array} $$ grâce à la positivité de $g$ et celle de $\sin$ sur $[0,\pi]$.

($\star\star$) Correction

Soit $f$ une fonction de classe $\CC^1$ sur $\R^+$, monotone et ayant une limite finie en $+\infty$.

  1. Montrer que les solutions de l'équation différentielle $(E)\ :\quad y''+y=f$ sont bornées sur $\R^+$.
  2. Montrer que $(E)$ admet une unique solution $g$ admettant une limite finie en $+\infty$.
  3. Déterminer $g$. Que dire de $\lim\limits_{+\infty} g$ ?

Correction

  1. Voir exercice précédent, cette équation admet pour solution \begin{equation*} \phi(x)=\dsp\int_0^x f(t)\sin(x-t)\,\ud t +A\cos x+B\sin x,\quad A,B\in \R. \end{equation*} Or, grâce à une intégration par parties : \begin{equation*} \dsp\int_0^x f(t)\sin (x-t)\,\ud t = f(x)-f(0)\cos x-\dsp\int_0^x f'(t)\cos(x-t)\,\ud t \end{equation*} et de plus $f$ est bornée sur $\R^+$. Comme $f'$ est de signe constant, l'intégrale $\dsp\int_0^{+\infty}\abs{f'(t)}\,\ud t $ est bien définie et donc $t\mapsto f'(t)\cos (x-t)$ est intégrable et $$ \abs{\dsp\int_0^{+\infty} f'(t)\cos(x-t)\,\ud t} \leq \dsp\int_0^{+\infty} \abs{f'(t)}\,\ud t \leq 2\norme{f}_\infty. $$ La fonction $\phi$ est donc bien bornée.
  2. De plus on peut écrire $$ \phi(x)=f(x)+ \left(A-f(0)-\dsp\int_0^x f'(t)\cos t\,\ud t\right)\cos x + \left(B-\dsp\int_0^x f'(t)\sin t\,\ud t\right)\sin x, $$ donc pour que $\phi$ admette une limite finie en~$+\infty$, il faut annuler les deux coefficients en $x\to+\infty$, donc il faut que \begin{equation*} A=f(0)+\dsp\int_{0}^{+\infty} f'(t)\cos t\,\ud t \quad \text{et } \quad B=\dsp\int_{0}^{+\infty} f'(t)\sin t\,\ud t. \end{equation*}
  3. On obtient alors $g(x)=f(x)+\dsp\int_x^{+\infty} f'(t)\,\cos(x-t)\,\ud t$ et par la construction précédente, le deuxième terme tend vers~$0$.
  4. Ainsi, $\lim\limits_{x\to+\infty} g(x)=\lim\limits_{x\to+\infty} f(x)$.

(Mines-Télécom PSI 2018) Correction

Soit $(E) : (2x+1)y'' + (4x-2)y' - 8y = 0$. Résoudre cette équation différentielle sur $]-\frac 12,+\infty[$, puis sur l'ensemble des réels.
(On déterminera d'abord une solution polynomiale, puis une solution de la forme $x\mapsto \ee^{ax}$, où $a$ est un réel).

Correction

On vérifie que $f(x)=x^2+\frac{1}{4}$ et $g(x)=\ee^{-2x}$ sont des solutions de $(E)$.